Browse Source

ÜB 4, Aufgabe 1 mit Lösung hinzugefügt

Martin Thoma 11 years ago
parent
commit
25fc7443b1

BIN
documents/GeoTopo/GeoTopo.pdf


+ 1 - 1
documents/GeoTopo/Kapitel1.tex

@@ -836,7 +836,7 @@ $\qed$
     gibt mit $\gamma(0)=x$ und $\gamma(1)=y$.
 \end{definition}
 
-\begin{korollar}
+\begin{korollar}\label{kor:wegzusammehang-impliziert-zusammenhang}
     Sei $X$ ein topologischer Raum.
 
     \begin{enumerate}[label=(\roman*)]

+ 12 - 0
documents/GeoTopo/Kapitel2-UB.tex

@@ -5,3 +5,15 @@
 \begin{aufgabe}\label{ub3:aufg1}
     \todo{Todo}
 \end{aufgabe}
+
+\begin{aufgabe}[Zusammenhang]\label{ub4:aufg1}
+    \begin{enumerate}[label=(\alph*)]
+        \item Beweisen Sie, dass eine topologische Mannigfaltigkeit
+              genau dann wegzusammenhängend ist, wenn sie zusammenhängend
+              ist
+        \item Betrachten Sie nun wie in Beispiel~\ref{bsp:mannigfaltigkeit8}
+              den Raum $X:= (\mdr \setminus \Set{0}) \cup \Set{0_1, 0_2}$
+              versehen mit der dort definierten Topologie. Ist $X$
+              wegzusammenhängend?
+    \end{enumerate}
+\end{aufgabe}

+ 7 - 7
documents/GeoTopo/Kapitel2.tex

@@ -87,20 +87,20 @@ U_i = \Set{(x_0: \dots : x_n) \in \praum^n(\mdr) | x_i \neq 0} &\rightarrow \mdr
               Zusammenhangskomponenten, wenn man einen Punkt entfernt.
         \item $V_2 = \Set{(x,y) \in \mdr^2 | x^3 = y^2}$ ist eine
               Mannigfaltigkeit.
-        \item $X = (\mdr \setminus \Set{0}) \cup (O_1, O_2)$
+        \item $X = (\mdr \setminus \Set{0}) \cup (0_1, 0_2)$ \label{bsp:mannigfaltigkeit8}
 
               \[U \subseteq X \text{ offen } \gdw 
                 \begin{cases}
-                    U \text{ offen in } \mdr \setminus \Set{0}, &\text{falls } O_1 \notin U, O_2 \in U\\
-                    \exists \varepsilon > 0 \text{ mit } (-\varepsilon, \varepsilon) \subseteq U &\text{falls } O_1 \in U, O_2 \in U
+                    U \text{ offen in } \mdr \setminus \Set{0}, &\text{falls } 0_1 \notin U, 0_2 \in U\\
+                    \exists \varepsilon > 0 \text{ mit } (-\varepsilon, \varepsilon) \subseteq U &\text{falls } 0_1 \in U, 0_2 \in U
                 \end{cases}\]
-              Insbesondere sind $(\mdr \setminus \Set{0}) \cup \Set{O_1}$
-              und $(\mdr \setminus \Set{0}) \cup \Set{O_2}$ offen und
+              Insbesondere sind $(\mdr \setminus \Set{0}) \cup \Set{0_1}$
+              und $(\mdr \setminus \Set{0}) \cup \Set{0_2}$ offen und
               homöomorph zu $\mdr$.
 
               \underline{Aber:} $X$ ist nicht hausdorffsch!
-              Denn es gibt keine disjunkten Umgebungen von $O_1$ und
-              $O_2$.
+              Denn es gibt keine disjunkten Umgebungen von $0_1$ und
+              $0_2$.
         \item $\GL_n(\mdr)$ ist eine Mannigfaltigkeit der Dimension 
               $n^2$, weil offene Teilmengen von $\mdr^{n^2}$ eine
               Mannigfaltigkeit bilden.

+ 59 - 0
documents/GeoTopo/Loesungen.tex

@@ -95,3 +95,62 @@
               \end{beweis}
     \end{enumerate}
 \end{solution}
+
+\begin{solution}[\ref{ub4:aufg1}]
+    \begin{enumerate}[label=(\alph*)]
+        \item \textbf{Vor.:} Sei $M$ eine topologische Mannigfaltigkeit.\\
+              \textbf{Beh.:} $M$ ist wegzusammehängend $\gdw M$ ist zusammenhängend
+              \begin{beweis}
+                \enquote{$\Rightarrow$}: Da $M$ insbesondere ein
+                topologischer Raum ist folgt diese Richtung direkt 
+                aus Korollar~\ref{kor:wegzusammehang-impliziert-zusammenhang}.
+
+                \enquote{$\Leftarrow$}: Seien $x,y \in M$ und
+                \[Z := \Set{z \in M | \exists \text{Weg von } x \text{ nach } z}\]
+                Es gilt:
+                \begin{enumerate}[label=(\roman*)]
+                    \item $Z \neq \emptyset$, da $M$ lokal wegzusammenhängend ist
+                    \item $Z$ ist offen, da $M$ lokal wegzusammenhängend ist
+                    \item $Z^C := \Set{\tilde{z} \in M | \nexists \text{Weg von } x \text{ nach } \tilde{z}}$ ist offen
+
+                    Da $M$ eine Mannigfaltigkeit ist, existiert zu jedem
+                    $\tilde{z} \in Z^C$ eine offene und wegzusammenhängende Umgebung 
+                    $U_{\tilde{z}} \subseteq M$.
+
+                    Es gilt sogar $U_{\tilde{z}} \subseteq Z^C$, denn
+                    gäbe es ein $U_{\tilde{z}} \ni \overline{z} \in Z$,
+                    so gäbe es Wege $\gamma_2:[0,1] \rightarrow M, \gamma_2(0) = \overline{z}, \gamma_2(1) = x$
+                    und $\gamma_1:[0,1] \rightarrow M, \gamma_1(0) = \tilde{z}, \gamma_1(1) = \overline{z}$.
+                    Dann wäre aber
+                    \[\gamma:[0,1] \rightarrow M,\;\;\; \gamma(x) = \begin{cases}
+                        \gamma_1(2x)   &\text{falls } 0 \leq x \leq \frac{1}{2}\\
+                        \gamma_2(2x-1) &\text{falls } \frac{1}{2} < x \leq 1
+                        \end{cases}\]
+                    ein stetiger Weg von $\tilde{z}$ nach $x$
+                    $\Rightarrow$ Widerspruch.
+
+                    Da $M$ zusammenhängend ist und $M = \underbrace{Z}_{\mathclap{\text{offen}}} \cup \underbrace{Z^C}_{\mathclap{\text{offen}}}$,
+                    sowie $Z \neq \emptyset$ folgt $Z^C = \emptyset$.
+                    Also ist $M=Z$ wegzusammenhängend.$\qed$
+                \end{enumerate}
+              \end{beweis}
+        \item \textbf{Beh.:} $X$ ist wegzusammenhängend.\\
+            \begin{beweis}
+                $X:= (\mdr \setminus \Set{0}) \cup \Set{0_1, 0_2}$
+                und $(\mdr \setminus \Set{0}) \cup \Set{0_2}$ sind
+                homöomorph zu $\mdr$. Also sind die einzigen kritischen
+                Punkte, die man nicht verbinden können könnte
+                $0_1$ und $0_2$.
+
+                Da $(\mdr \setminus \Set{0}) \cup \Set{0_1}$ homöomorph
+                zu $\mdr$ ist, exisitert ein Weg $\gamma_1$ von $0_1$
+                zu einem beliebigen Punkt $a \in \mdr \setminus \Set{0}$.
+                
+                Da $(\mdr \setminus \Set{0}) \cup \Set{0_2}$ ebenfalls
+                homöomorph zu $\mdr$ ist, existiert außerdem ein Weg
+                $\gamma_2$ von $a$ nach $0_2$. Damit existiert ein
+                (nicht einfacher)
+                Weg $\gamma$ von $0_1$ nach $0_2$. $\qed$
+            \end{beweis}
+    \end{enumerate}
+\end{solution}